Find the minimum possible value of k such that the following inequality holds for all triples of positive real numbers ( x , y , z ) : x + y + z + k ≥ 2 x + 2 y + 2 z
This section requires Javascript.
You are seeing this because something didn't load right. We suggest you, (a) try
refreshing the page, (b) enabling javascript if it is disabled on your browser and,
finally, (c)
loading the
non-javascript version of this page
. We're sorry about the hassle.
Oops I forgot to mention: through the equality cases for Cauchy-Schwarz and AM-GM, we can find that equality only holds for x = y = z = 1 .
Let x = a 2 , y = b 2 , z = c 2 . The inequality in question can be re-arranged as: ( a 2 − 2 a ) + ( b 2 − 2 b ) + ( c 2 − 2 c ) ≥ − k Observe that a 2 − 2 a = ( a − 1 ) 2 − 1 This shows that the minimum possible value of a 2 − 2 a is − 1 , which is attained for a = 1 . Similarly, b 2 − 2 b ≥ − 1 c 2 − 2 c ≥ − 1 Adding them, ( a 2 − 2 a ) + ( b 2 − 2 b ) + ( c 2 − 2 c ) ≥ − 3 Equality is attained for ( a , b , c ) = ( 1 , 1 , 1 ) . We thus conclude our answer is 3 .
I did it by differentiating the function f ( x ) = x − 2 x and finding at which point it gives 0 ,thus finding the minimum of the function.It turned out that the minimum is when x=1 and is equal to -1.Since there are three such sums for x,y and z, so the minimum of c y c ∑ x − 2 x is 3.(-1)=-3.So k=3!
Let f ( x , y , z ) = 2 x − x + 2 y − y + 2 z − z . If we take g r a d f = 0 , then we obtain:
f x = x 1 − 1 = 0 , f y = y 1 − 1 = 0 , f z = z 1 − 1 = 0 ⇒ x = y = z = 1 (i).
The Hessian Matrix at this critical point evaluates to:
F ( x , y , z ) = ⎣ ⎡ f x x f y x f z x f x y f y y f z y f x z f y z f z z ⎦ ⎤ = ⎣ ⎡ − 2 x 3 / 2 1 0 0 0 − 2 y 3 / 2 1 0 0 0 − 2 z 3 / 2 1 ⎦ ⎤ ;
or F ( 1 , 1 , 1 ) = − 2 1 ⋅ I 3 x 3 , which is negative-definite (hence, P ( 1 , 1 , 1 ) is the global maximum point of f over x , y , z ∈ R + ). Thus, k ≥ f ( 1 , 1 , 1 ) = 3 ( 2 1 − 1 ) ⇒ k ≥ 3 .
Problem Loading...
Note Loading...
Set Loading...
First, notice that plugging in x = y = z = 1 gives 3 + k ≥ 6 , so k ≥ 3 . Now, we will prove that 3 is indeed the minimum.
By AM-GM, 2 x + y + z + 3 = 2 ( x + 2 ) + ( y + z + 1 ) ≥ ( x + 2 ) ( y + z + 1 ) By Cauchy-Schwarz, ( x + 2 ) ( y + z + 1 ) = ( x + 1 + 1 ) ( 1 + y + z ) ≥ ( x + y + z ) 2 and plugging into the inequality from AM-GM, 2 x + y + z + 3 ≥ x + y + z which simplifies to the desired inequality.
Motivation: Having multiple square roots on the lesser side is troublesome, a single one could be obtained by AM-GM, but we also realize that Cauchy-Schwarz, in effect, takes square roots. Then, we must obtain another square root, which is possible through AM-GM.